LSAT and Law School Admissions Forum

Get expert LSAT preparation and law school admissions advice from PowerScore Test Preparation.

User avatar
 Dave Killoran
PowerScore Staff
  • PowerScore Staff
  • Posts: 5852
  • Joined: Mar 25, 2011
|
#43244
Setup and Rule Diagram Explanation

This is an Advanced Linear: Balanced game.

The game scenario indicates that six tasks will be demonstrated at a farm exhibition. Each task is demonstrated one after the other, giving the game a Linear aspect, and each demonstration is given by one of three volunteers. The initial scenario appears as follows:
PowerScore_LG_38_October_2002_game _#2_setup_diagram 1.png
The first rule, which involves F and G, has a powerful effect, and creates exactly two possible sequences for F and G:
PowerScore_LG_38_October_2002_game _#2_setup_diagram 2.png
Note that this eliminates G from demonstrating the first task. More on the limitations produced by this rule later.

The second rule creates F Not Laws on the first and last demonstrations, leaving only L available to demonstrate the first task, and G or L for the last task. With L demonstrating the first task, by applying the first rule a G Not Law can be placed on the second demonstration as well (creating a F/L dual-option):
PowerScore_LG_38_October_2002_game _#2_setup_diagram 3.png
The third and fourth rules are similar, and each removes two tasks from a volunteer:
PowerScore_LG_38_October_2002_game _#2_setup_diagram 4.png
When combined, these rules eliminate both G and L from harvesting, resulting in the inference that F must demonstrate harvesting:
PowerScore_LG_38_October_2002_game _#2_setup_diagram 5.png
And, because from the second rule F can perform neither first nor last, we can deduce that harvesting is demonstrated neither first nor last. In addition, because L must demonstrate the first task, T cannot be demonstrated first (H has already been eliminated):
PowerScore_LG_38_October_2002_game _#2_setup_diagram 6.png
The fifth and final rule creates a standard block:
PowerScore_LG_38_October_2002_game _#2_setup_diagram 11.png
The application of this block eliminates T from being demonstrated last, M from being demonstrated first and second (remember, L already demonstrates the first task and L cannot demonstrate T from the fourth rule, which means that the earliest T could be demonstrated is second). Adding in the randoms, we near the final setup:
PowerScore_LG_38_October_2002_game _#2_setup_diagram 7.png
Before moving on to the questions, let’s revisit the possible orderings of F, G, and L. The first rule, which involves F and G, has a controlling effect on the performances of the three volunteers. As stated earlier, the rule creates exactly two possible sequences for F and G:
PowerScore_LG_38_October_2002_game _#2_setup_diagram 8.png

Because, as discussed during the setup, L must perform first, the only wild card in the two sequences above is L’s second performance. In the case of Sequence 1, because F cannot perform last, L must perform last, producing just one acceptable ordering of the volunteers:
PowerScore_LG_38_October_2002_game _#2_setup_diagram 9.png
In the case of Sequence 2, L’s second performance can be second, third, fourth, fifth, or sixth, producing five acceptable orderings of the volunteers:
PowerScore_LG_38_October_2002_game _#2_setup_diagram 10.png
Although these six orders limit the possibilities in the game, there are too many combinations of the tasks-to-volunteers to make it worthwhile to Identify the Possibilities or Templates.
You do not have the required permissions to view the files attached to this post.
 SherryZ
  • Posts: 124
  • Joined: Oct 06, 2013
|
#12016
Thank you very much for your help!

Oct 2002 LSAT Sec 2, Game #2:

Could you show me how to set up in this game correctly?? I spent 20min to do it :(

Also, could you explain Q11? I chose C but the right answer is A.

Thank you very much!

---Sherry
User avatar
 Dave Killoran
PowerScore Staff
  • PowerScore Staff
  • Posts: 5852
  • Joined: Mar 25, 2011
|
#12019
Hi Sherry,

This is a somewhat complicated setup that involves several pages and a lot of Not Laws, so I'm going to refer you to the October 2002 Logic Setups on this page if you are looking for the complete solution to this game (as well as the rest of the games in this section).

The explanation to question #11 is beyond the capacity of this forum to diagram (it really is almost a work of art :-D with all the blocks and arrows ), so I'll refer you to that same file for the complete solution. But, in that question, the only two possible performance orderings of volunteers are:


..... ..... ..... L-F-G-G-F-L
..... ..... or
..... ..... ..... L-F-G-F-L-G


In both scenarios, F will demonstrate harvesting second, and thus answer choice (A) is correct.

Thanks!
 kcho10
  • Posts: 69
  • Joined: Nov 02, 2015
|
#20475
Hi,
Can someone please explain how we come to diagram Rule #1 as F--G--G--F or F--G--F--G?

To me, it seems like the second hypothetical breaks the rule because the second G has 2 F's before it, yet the rule says that Frank demonstrates exactly one task before Gladys demonstrates ANY of the tasks. Wouldn't the any trigger a sufficient condition, making the statement out to be

G--> F..G..F

But then, for the second hypothetical, it would be F...F...G, and wouldn't that break the rule?

Thank you!
 Ricky_Hutchens
PowerScore Staff
  • PowerScore Staff
  • Posts: 59
  • Joined: Oct 12, 2015
|
#20490
Hi,

That's a good question and I understand why you read the sentence that way. But I think your interpretation would be better suited if the statement was "F must perform exactly once before any task that is performed by G." This would force all of G's tasks between F's because we are talking specifically about "any task performed by G."

Here we have "F must perform exactly once before G performs any task." The "any" in this case modifies the generic task and is not limited as it is in the above example. It is there to tell us G can not perform until F has performed exactly once. This rule has two affects:

1) We know that F's first performance is before G's first performance
2) We know that F's second performance is after G's first performance.

Notice that once F performs once before G's first performance, the rule is satisfied completely and we know nothing about the relationship between F and G's second performances.

Hope that helps.
 nutcracker
  • Posts: 39
  • Joined: Aug 13, 2017
|
#43211
Hello! The link that Dave provided above doesn't seem to work anymore. Could anyone reset it so we can all learn from the genius setup? :-D Thank you very much!
 Administrator
PowerScore Staff
  • PowerScore Staff
  • Posts: 8917
  • Joined: Feb 02, 2011
|
#43245
Hi,

We've placed the official setup at the top of this thread!
User avatar
 fortunateking
  • Posts: 31
  • Joined: Jan 10, 2022
|
#96573
Dear PS people,
When working on this question, I interpret the first rule's "one task before" as "Fs sit exactly one space before any of the 2 Gs", which means FGFG.
Can you tell me why this is wrong?
 Adam Tyson
PowerScore Staff
  • PowerScore Staff
  • Posts: 5153
  • Joined: Apr 14, 2011
|
#97251
You're treating that rule as being about specific order, fortunateking, as if it said "Frank demonstrates one task immediately before Gladys." But it's actually about quantity rather than specific order, and just means that F goes "exactly one" time before G. "Exactly one task" is a quantifier, as opposed to "immediately before" which is about placement.

Also, note the use of "any" in regard to G. In order for the rule to be interpreted the way you did, it would have to say "before each" rather than "before any." For example, "F demonstrates one task immediately before each task G demonstrates." Instead, F going once before any G means there is one F before the first G, and the second F has to be somewhere after the first G, regardless of the specific order after that first G.

Get the most out of your LSAT Prep Plus subscription.

Analyze and track your performance with our Testing and Analytics Package.